Đến nội dung

MrS nội dung

Có 33 mục bởi MrS (Tìm giới hạn từ 02-05-2020)



Sắp theo                Sắp xếp  

#658514 Topic: [LTDH] Mỗi ngày hai bất đẳng thức.

Đã gửi bởi MrS on 20-10-2016 - 11:11 trong Bất đẳng thức và cực trị

Tiếp theo:

Bài 105: Cho ba số thực dương $a,b,c$ thỏa mãn: $a+b+c=3$. Tìm GTNN của biểu thức: $P=\sum \frac{b\sqrt{b}}{\sqrt{2a+b+c}}$.

$\sum \frac{b\sqrt{b}}{\sqrt{2a+b+c}}= \sum \frac{2b^2}{\sqrt{4b(2a+b+c)}}\geq \sum \frac{4b^2}{2a+5b+c}\geq \frac{4(a +b+c)^2}{8(a+b+c)}=\frac{3}{2}$




#658018 Topic: [LTDH] Mỗi ngày hai bất đẳng thức.

Đã gửi bởi MrS on 16-10-2016 - 03:31 trong Bất đẳng thức và cực trị

Tiếp theo: 

Bài 98: Cho ba số thực dương $a,b,c$ thỏa mãn: $ab+bc+ca=3abc$. Tìm GTNN của biểu thức:

$P=(\frac{a}{b}-\frac{1}{b}+\frac{1}{ab})(\frac{b}{c}-\frac{1}{c}+\frac{1}{bc})(\frac{c}{a}-\frac{1}{a}+\frac{1}{ca})$

 

Bài 98: Đổi: $(\frac{1}{a}-1,\frac{1}{b}-1,\frac{1}{c}-1)\rightarrow (x,y,z)$

Khi đó $P=(x^2+x+1)(y^2+y+1)(z^2+z+1)$ với $x+y+z=0$

Thế $z=-x-y$ vào P ta sẽ CM: $(x^2+x+1)(y^2+y+1)((x+y)^2-x-y+1)-1\geq 0$ (*)

Đặt $x+y=S; xy =P$ BĐT trên trở thành: $f(P)=(S^2-S+1)P^2+(S-1)(S^2-S+1)P+S^4+S^2\geq 0$

Không mất tổng quá, giả sử $P=xy\geq 0$ lại có $z>-1\Rightarrow S=x+y=-z<1$ và $S^2\geq 4P$

Ta có: $f"(P)=2(S^2-S+1)>0\Rightarrow f'(P)\geq f'(0)=(S-1)(S^2-S+1)<0\Rightarrow f(P)\geq f(\frac{S^2}{4})=\frac{S^2}{16}((S^2+\frac{3S}{2})^2+4(S+1)^2+\frac{11S^2}{4}+8)\geq 0$

(*) được CM. Vậy $MinP=1$ tại $a=b=c=1$




#658016 Topic: [LTDH] Mỗi ngày hai bất đẳng thức.

Đã gửi bởi MrS on 16-10-2016 - 01:04 trong Bất đẳng thức và cực trị

Tiếp theo: 

Bài 97: Cho $x,y,z$ là các số thực dương. Chứng minh rằng:

$\frac{2x^2+xy}{(y+\sqrt{zx}+z)^2}+\frac{2y^2+yz}{(z+\sqrt{xy}+x)^2}+\frac{2z^2+zx}{(x+\sqrt{yz}+y)^2}\ge 1$.

Bài 97:

Sử dụng Cauchy - Schwarz: $(y+\sqrt{zx}+z)^2\leq (y+z+z)(y+x+z)=(x+y+z)(y+2z)$

Từ đó ta chỉ cần chứng minh: $P=\sum \frac{2x^2+xy}{y+2z}\geq x+y+z$

Theo Cauchy - Schwarz: $P\geq \frac{(2\sum x^2+\sum xy)^2}{\sum (y+2z)(2x^2+xy)}=\frac{(2\sum x^2+\sum xy)^2}{2(xy+yz+zx)(x+y+z)+3(xy^2+yz^2+zx^2+xyz)-3xyz}=M$

Áp dụng bổ đề quen thuộc: $xy^2+yz^2+zx^2+xyz\leq \frac{4(x+y+z)^3}{27}$

Suy ra: $M\geq \frac{(2\sum x^2+\sum xy)^2}{2(xy+yz+zx)(x+y+z)+\frac{4(x+y+z)^3}{9}-3xyz}$

Đưa về dạng $p, q, r$ và chuẩn hóa $p=1$

Ta đi chứng minh: $\frac{(2-3q)^2}{2q+\frac{4}{9}-3r}\geq 1\Leftrightarrow 81q^2-126q+32+27r\geq 0$ (*)

Theo BĐT Schur thì: $r\geq \frac{p(4q-p^2)}{9}=\frac{4q-1}{9}$

Nên $VT(*)\geq (1-3q)(29-27q)\geq 0$ hiển nhiên đúng do $0< q\leq \frac{1}{3}< \frac{29}{27}$

Bài toán được chứng minh. Đẳng thức xảy ra khi  $x=y=z$




#657741 Topic: [LTDH] Mỗi ngày hai bất đẳng thức.

Đã gửi bởi MrS on 13-10-2016 - 16:39 trong Bất đẳng thức và cực trị

 

Bài 94: Cho $a,b,c$ là các số thực không âm thỏa mãn điều kiện: $a^2+b^2+c^2=1$. Tìm GTLN của: $P=(a-b)(b-c)(c-a)(a+b+c)$

Đổi thành a,c,b thưc thì là bài 3 của IMO 2006 :D 




#657641 Topic: [LTDH] Mỗi ngày hai bất đẳng thức.

Đã gửi bởi MrS on 12-10-2016 - 18:39 trong Bất đẳng thức và cực trị

Bài 87: Royal1534 đã cho lời giải đúng. Dưới đây là lời giải bài 88:

Lời giải bài 88:

Ta có: $\sqrt{ab+1}\ge \sqrt{ab+\frac{(a-b)^2}{2}}=|\frac{a+b}{2}|$.

Tương tự ta được: $\sum \sqrt{ab+1}\ge \sum |\frac{a+b}{2}|\ge |a+b+c|\ge a+b+c$.

(Áp dụng: $|x|+|y|+|z|\ge |x+y+z|$).

Tiếp theo:

Bài 91: Cho các số thực dương $x,y,z$. Chứng minh rằng:

$\frac{1}{x}+\frac{1}{y}+\frac{1}{z}\ge \frac{36}{9+x^2y^2+y^2z^2+z^2x^2}$.

Bài 92: Cho các số thực dương $x,y,z$ thỏa mãn: $x+y+z\le 1$. Tìm GTNN của biểu thức:

$P=4x+3y+24z+\frac{19}{x}+\frac{25}{3y}+\frac{8}{3z}$. 

Bài 91: Theo AM - GM:  $(9+x^2y^2+y^2z^2+z^2x^2)(\frac{1}{x}+\frac{1}{y}+\frac{1}{z})\geq (9+3\sqrt[3]{(xyz)^4})(\frac{3}{\sqrt[3]{xyz}})\geq \frac{36\sqrt[4]{\sqrt[3]{(xyz)^4}}}{\sqrt[3]{xyz}}= 36$

 

Bài 92: Theo AM - GM: $P=19(4x+\frac{1}{x})+25(3y+\frac{1}{3y})+8(12z+\frac{1}{3z})-72(x+y+z)\geq 76+50+32-72=86.$

$\Rightarrow MinP=86$ tại $x=\frac{1}{2}, y=\frac{1}{3}, z= \frac{1}{6}$




#657376 Topic: [LTDH] Mỗi ngày hai bất đẳng thức.

Đã gửi bởi MrS on 10-10-2016 - 01:02 trong Bất đẳng thức và cực trị

Tiếp theo:

Bài 85: Cho các số thực dương $a,b,c$ thỏa mãn: $abc=1$. Chứng minh rằng:

$\sqrt{2(a^2+1)}+\sqrt{2(b^2+1)}+\sqrt{2(c^2+1)}\le 1+\frac{5}{3}(a+b+c)$.

Ta dễ dàng chứng minh được: $\sqrt{2(a^2+1)}\leq \frac{1+5a-2\cdot lna}{3}, \forall a>0$

Làm tương tự với $b,c$ ta suy ra đpcm.




#655756 CMR: $\sum \frac{(b+c-a)^2}{(b+c)^2+a^2}...

Đã gửi bởi MrS on 27-09-2016 - 18:05 trong Bất đẳng thức và cực trị

Nếu $x + y = 0$ thì trong hai số $x,y$ sẽ có một số $\geqslant 0.$ Mà

\[a+b+c+[(-a) +(-b)+(-c)] = 0.\]

;)

Từ trước tới nay em toàn làm máy móc và hiểu sai bản chất :((




#655752 Topic: [LTDH] Mỗi ngày hai bất đẳng thức.

Đã gửi bởi MrS on 27-09-2016 - 17:34 trong Bất đẳng thức và cực trị

Bài 59 và bài 60: Các bạn hanguyen445 và MrS đã cho lời giải đúng.

Dưới đây là cách khác của bài 60:

Ta có: $3(a^2+b^2+c^2)\ge (a+b+c)^2\implies a+b+c\le 3$.

Xét $\frac{1}{4-\sqrt{ab}}\le \frac{1}{4-\frac{a+b}{2}}=\frac{2}{4-a+4-b}\le \frac{1}{2}(\frac{1}{4-a}+\frac{1}{4-b})$.

Vì vậy: $P\le \frac{1}{4-a}+\frac{1}{4-b}+\frac{1}{4-c}$.

Đặt $M=\frac{1}{4-a}+\frac{1}{4-b}+\frac{1}{4-c}$.

Ta có: $3-M=\sum (1-\frac{1}{4-a})=\sum \frac{3-a}{4-a}$.

$\iff 3-M=\sum \frac{(3-a)^2}{(4-a)(3-a)}\ge \frac{(9-a-b-c)^2}{39-7(a+b+c)}$.

Ta đi xét hàm: $f(t)=\frac{(9-t)^2}{39-7t},0<t\le 3$.

Hàm $f(t)$ đồng biến trên $(0;3]$ nên $max(f(t))=2$.

$\implies P\ge 1(Q.E.D)$.

Phải tìm Minf(t) chứ tritanngo99 nhỉ :)




#655750 Topic: [LTDH] Mỗi ngày hai bất đẳng thức.

Đã gửi bởi MrS on 27-09-2016 - 17:28 trong Bất đẳng thức và cực trị

Tiếp theo: 

Bài 61: Cho $x,y,z$ là các số thực dương thỏa mãn: $xy\ge 1;z\ge 1$. Tìm GTNN của:

$P=\frac{x}{y+1}+\frac{y}{x+1}+\frac{z^3+2}{3(xy+1)}$.

Bài 61 :

Theo BĐT Cauchy - Schwazt, AM-GM:

$P\geq \frac{x}{y+1}+\frac{y}{x+1}+\frac{1}{xy+1}\geq \frac{(x+y)^2}{2xy+x+y}+\frac{1}{xy+1}\geq \frac{(2\sqrt{xy})^2}{2xy+2\sqrt{xy}}+\frac{1}{xy+1}=\frac{2\sqrt{xy}}{\sqrt{xy}+1}+\frac{1}{xy+1}=\frac{(\sqrt{xy}-1)^3}{2(\sqrt{xy}+1)(xy+1)}+\frac{3}{2}\geq \frac{3}{2}$ do $xy\ge 1;z\ge 1$

Vậy MinP=3/2, dấu bằng xảy ra khi x=y=z=1.




#655624 Topic: [LTDH] Mỗi ngày hai bất đẳng thức.

Đã gửi bởi MrS on 26-09-2016 - 17:25 trong Bất đẳng thức và cực trị

Tiếp theo: 

Bài 59: Cho $a,b,c>0$. Chứng minh rằng: $(a^5+\frac{2}{a})(b^5+\frac{2}{b})(c^5+\frac{2}{c})\ge (a+b+c)^3$.

Bài 60: Cho $a,b,c>0$ thỏa mãn: $a^2+b^2+c^2=3$. Chứng minh rằng:

$\frac{1}{4-\sqrt{ab}}+\frac{1}{4-\sqrt{bc}}+\frac{1}{4-\sqrt{ca}}\le 1$

Bài 59: 

Theo BĐT Holder: $(a^5+\frac{2}{a})(b^5+\frac{2}{b})(c^5+\frac{2}{c})\ge (\sqrt[3]{\frac{a^5}{bc}}+\sqrt[3]{\frac{b^5}{ca}}+\sqrt[3]{\frac{c^5}{ab}})^3$

Từ đó chỉ cần chứng minh: $\sqrt[3]{\frac{a^5}{bc}}+\sqrt[3]{\frac{b^5}{ca}}+\sqrt[3]{\frac{c^5}{ab}}\geq a+b+c$

Có nhiều cách CM BĐT này. 

Chuẩn hóa abc=1 khi đó ta cần chứng minh: $\sum a^2\geq \sum a$

Theo AM-GM: $a^2+b^2+c^2\geq \frac{1}{3}(a+b+c)^2, a^2+b^2+c^2\geq3\sqrt[3]{a^2b^2c^2}=3\Rightarrow (a^2+b^2+c^2)^2\geq (a+b+c)^2\Rightarrow \sum a^2\geq \sum a$

Từ đó ta có đpcm. Đẳng thức xảy ra khi a=b=c=1.

 

Bài 60: Theo BĐT AM-GM: $\sum \frac{1}{4-\sqrt{ab}}\leq \sum \frac{1}{4-\sqrt{\frac{a^2+b^2}{2}}}=\sum \frac{1}{4-\sqrt{\frac{3-c^2}{2}}}$

Ta sẽ chứng minh: $\frac{1}{4-\sqrt{\frac{3-x^2}{2}}}\leq \frac{13-x^2}{36}$ (*)

Tới đây chỉ cần đặt: $\sqrt{\frac{3-x^2}{2}}=t \Rightarrow t\in (0,\sqrt{\frac{3}{2}})$

BĐT (*) trở thành: $\frac{1}{4-t}\leq \frac{5+t^2}{18}\Leftrightarrow (t-1)^2(2-t)\geq 0$ luôn đúng do $t\in (0,\sqrt{\frac{3}{2}})$

Từ đó dễ dàng suy ra: $\sum \frac{1}{4-\sqrt{ab}}\leq \sum \frac{13-a^2}{36}=1$

Đẳng thức xảy ra khi a=b=c=1.




#655586 CMR: $\sum \frac{(b+c-a)^2}{(b+c)^2+a^2}...

Đã gửi bởi MrS on 26-09-2016 - 11:52 trong Bất đẳng thức và cực trị

Đúng rồi em. :)

Anh ơi em có vài thắc mắc anh giúp em với :)

"Từ việc thay (a,b,c) bởi (-a,-b,-c) mà BĐT không đổi tại sao lại chỉ cần xét khi a,b,c không âm nhỉ". Nếu đẳng thức xảy là khi có cả số âm, cả không âm thì sao nhỉ? 




#655568 Topic: [LTDH] Mỗi ngày hai bất đẳng thức.

Đã gửi bởi MrS on 25-09-2016 - 22:47 trong Bất đẳng thức và cực trị

Hoán vị vòng quanh mà bạn. Nên cần xét thêm trường hợp nữa. 

 \[BDT \Leftrightarrow 2{\left( {a + b + c} \right)^3} \geqslant 6\left( {a + b + c} \right)\left( {ab + bc + ac} \right) + a{\left( {a - b} \right)^2} + b{\left( {b - c} \right)^2} + c{\left( {c - a} \right)^2}\]

\[ \Leftrightarrow \left( {a + b + c} \right)\sum {{{\left( {a - b} \right)}^2}}  \geqslant a{\left( {a - b} \right)^2} + b{\left( {b - c} \right)^2} + c{\left( {c - a} \right)^2}\]

\[ \Leftrightarrow \left( {b + c} \right){\left( {a - b} \right)^2} + \left( {a + c} \right){\left( {b - c} \right)^2} + \left( {a + b} \right){\left( {a - c} \right)^2} \geqslant 0\




#655542 Topic: [LTDH] Mỗi ngày hai bất đẳng thức.

Đã gửi bởi MrS on 25-09-2016 - 20:30 trong Bất đẳng thức và cực trị

Tiếp theo:

Bài 57: Cho $a,b,c$ là các số thực dương thỏa mãn: $a+b+c=3$. Chứng minh rằng:

$18(ab+bc+ca)+a(a-b)^2+b(b-c)^2+c(c-a)^2\le 54$.

BĐT tương đương: $6(ab+bc+ca)(a+b+c)+a(a-b)^2+b(b-c)^2+c(c-a)^2\le 2(a+b+c)^3\Leftrightarrow \sum a^3+2\sum a^2b\geqslant \sum ab^2+6abc$

Giả sử $a\geq b\geq c$ và theo BĐT AM-GM: $\sum a^3+\sum ab^2\geqslant 2\sum a^2b$ và $\sum a^2b\geq 3abc$ 

Mặt khác: $(a-b)(b-c)(a-c)\geq 0\Rightarrow \sum a^2b\geq \sum ab^2$

Từ các BĐT trên suy ra đpcm.

Đẳng thức xảy ra khi a=b=c=1




#655532 Topic: [LTDH] Mỗi ngày hai bất đẳng thức.

Đã gửi bởi MrS on 25-09-2016 - 19:37 trong Bất đẳng thức và cực trị

Cảm ơn anh nhiều MrS về bài 49, Thành thật xin lỗi mọi người vì những thiếu sót này. Lần sau mình cố gắng cẩn thận hơn. Mong mọi người thông cảm.

Bài 52  xin đưa ra lời giải như sau:

Theo BDT Cauchy-Schwarz ta có: $(x+y+z)^2\le 6(x^2+\frac{y^2}{2}+\frac{z^2}{3})\le 36$.

$\implies x+y+z\in [-6;6]$.

Đặt $t=x+y+z,t\in [-6;6]$.

Xét hàm số: $f(t)=\frac{t^3-36}{t},t\in [-6;6]$

$\implies f'(t)=2t+\frac{36}{t^2}$.

$f'(t)=0\iff t=-\sqrt[3]{18}$.

$f(-6)=42,f(-\sqrt[3]{18})=\frac{54}{\sqrt[3]{54}},f(6)=30\implies f(t)(Max)=42$.

Dấu $=$ xảy ra tại $x=-1;y=-2;z=-3$.

Vậy $Max(P)=42$.

Bạn quên mất là f(t) bị gián đoạn tại t=0 kìa :)

Khi t->0- thì f(t) -> +vô cực. Tức là khả năng đề bài không đúng :)




#655220 Topic: [LTDH] Mỗi ngày hai bất đẳng thức.

Đã gửi bởi MrS on 23-09-2016 - 11:27 trong Bất đẳng thức và cực trị

Tiếp theo:

Bài 52: Cho các số thực $x,y,z$ thỏa mãn: $x^2+\frac{y^2}{2}+\frac{z^2}{3}=6$. Tìm GTLN của biểu thức:

$P=\frac{(x+y+z)^3-36}{x+y+z}$

Theo Cauchy - Scharzt: $(x^2+\frac{y^2}{2}+\frac{z^2}{3})(1+2+3)\geq (x+y+z)^2\Rightarrow -6\leq x+y+z\leq 6$

Xét $f(t)=\frac{t^3-36}{t}=t^2-\frac{36}{t}, -6\leq t\leq 6$

Tới đây mình tịt :D vì khi t->0- thì f(t)-> +vô cực.

Chắc thêm giả thiết x+y+z>0 thì giải được :D

Không biết đáp án thế nào.




#655215 Topic: [LTDH] Mỗi ngày hai bất đẳng thức.

Đã gửi bởi MrS on 23-09-2016 - 10:29 trong Bất đẳng thức và cực trị

Tiếp theo:

Bài 51: Cho các số thực dương $x,y,z$. Tìm GTNN của biểu thức:

 $P=\frac{1}{6\sqrt{xy}+7z+8\sqrt{zx}}-\frac{1}{9\sqrt{x+y+z}}$.

Theo AM-GM: $6\sqrt{xy}+7z+8\sqrt{zx}\leq (x+9y)+7z+2(z+4x)=9(x+y+z)\Rightarrow P\geq \frac{1}{9}(\frac{1}{\sqrt{x+y+z}}-\frac{1}{2})^2-\frac{1}{36}\geq -\frac{1}{36}$

Vậy $MinP=-\frac{1}{36}$, đạt được khi $x=\frac{18}{23}, y=\frac{2}{23}, z=\frac{72}{23}$




#655214 Topic: [LTDH] Mỗi ngày hai bất đẳng thức.

Đã gửi bởi MrS on 23-09-2016 - 09:53 trong Bất đẳng thức và cực trị

 

Dưới đây là lời giải bài 49 và bài 50:

 

Lời giải bài 49: Chuẩn hóa: $a^2+b^2+c^2=9$.

 

Ta có: $BDT\iff 2(a+b+c)-abc\le 10$.

 

$VT=2(a+b+c)-abc=2a-abc+2(b+c)=a(2-bc)+2(b+c)$

 

$VT^2\le [a^2+(b+c)^2][(2-bc)^2+4]$.

 

KMTTQ, giả sử: $a\ge b\ge c\implies a^2\ge 3$.

 

Đặt $t=bc\implies t=bc\le \frac{b^2+c^2}{2}=\frac{9-a^2}{2}\le 3$.

 

Nên $VT^2\le (9+2bc)[(2-bc)^2+4]=(9+2t)[(2-t)^2+4]=f(t)\text{   }\forall t\in(0;3]$.

 

Khảo sát $f(t)\implies f(t)\le 100\implies VT\le 10\implies Q.E.D$.

 

Dấu $=$ xảy ra tại $a=b=c$.

 

Không có đẳng thức vì $f(t)=100\Leftrightarrow (t+2)(t^2-3t-14)=0$ :D




#655071 Topic: [LTDH] Mỗi ngày hai bất đẳng thức.

Đã gửi bởi MrS on 22-09-2016 - 00:12 trong Bất đẳng thức và cực trị

Bài tập 49:

Do BĐT thuần nhất nên chuẩn hóa $a+b+c=3$

  \[ \to 2{\left( {a + b + c} \right)^2} > {a^2} + {b^2} + {c^2} \ge \frac{{{{\left( {a + b + c} \right)}^2}}}{3}{\rm{ hay }}{a^2} + {b^2} + {c^2} \in \left[ {0;1} \right)\]

\[ \Rightarrow \exists t \in \left[ {0;1} \right]:{a^2} + {b^2} + {c^2} = 3 + 6{t^2} \to ab + bc + ac = 3 - 3{t^2}\]

 

Sao lại có điều này được nhỉ?

Ngoài ra trong bài này chỉ xảy ra đẳng thức khi (x,y,z)=(-1,2,2) hoặc hoán vị. Trong bài này thì không có dấu bằng.

Bài này có thể tham khảo VD4 thầy Dũng viết :) :http://diendantoanho...hức-thuần-nhất/




#655046 Topic: [LTDH] Mỗi ngày hai bất đẳng thức.

Đã gửi bởi MrS on 21-09-2016 - 21:47 trong Bất đẳng thức và cực trị

 

Tiếp theo:

Bài 49: Cho $a,b,c>0$. Chứng minh rằng: $6(a+b+c)(a^2+b^2+c^2)\le 27abc+10(a^2+b^2+c^2)^{\frac{3}{2}}$.

 

 

Bạn xem trong này nhé, nó đúng cả trong trường hợp a,b,c thực.

http://diendantoanho...hức-thuần-nhất/




#655028 Topic: [LTDH] Mỗi ngày hai bất đẳng thức.

Đã gửi bởi MrS on 21-09-2016 - 20:25 trong Bất đẳng thức và cực trị

Tiếp theo:

Bài 49: Cho $a,b,c>0$. Chứng minh rằng: $6(a+b+c)(a^2+b^2+c^2)\le 27abc+10(a^2+b^2+c^2)$.

 

Đề bài sai hay sao ý :D: cho $a=10^{-7}, b=c=1 \Rightarrow VP-VT\approx -4< 0$




#654961 CMR: $\sum \frac{(b+c-a)^2}{(b+c)^2+a^2}...

Đã gửi bởi MrS on 21-09-2016 - 01:15 trong Bất đẳng thức và cực trị

Để chuẩn hóa $a+b+c=3$ thì em cần phải có $a+b+c>0.$ Nhưng điều này ta có thể giả sử được, em thử nghĩ xem nhé.

Trước em có đọc 1 vài tài liệu thì người ta nói rằng: Nếu thay bộ (a,b,c) bởi (-a,-b,-c) BĐT không đổi thì có thể giả sử a+b+c > 0. Không  biết có đúng không ạ? 




#654955 CMR: $\sum \frac{(b+c-a)^2}{(b+c)^2+a^2}...

Đã gửi bởi MrS on 20-09-2016 - 23:37 trong Bất đẳng thức và cực trị

Lời giải của MrS sai ngay từ đây.

Phải xét a+b+c=0 nữa hả anh ơi?  :(




#654897 Topic: [LTDH] Mỗi ngày hai bất đẳng thức.

Đã gửi bởi MrS on 20-09-2016 - 18:28 trong Bất đẳng thức và cực trị

Bài 48:

Không mất tổng quát giả sử c=min(a, b ,c).

Trước tiên ta CM bổ đề quen thuộc: $\sqrt{\frac{a}{b+c}}+\sqrt{\frac{b}{c+a}}\geq 2\sqrt{\frac{a+b}{a+b+2c}}$ ( có thể dùng Holder để chứng minh)

Lại theo AM - GM: $\sqrt{ab}+\sqrt{bc}+\sqrt{ca}\leq \frac{a+b}{2}+\sqrt{2c(a+b)}$

Đặt: $\sqrt{\frac{2c}{b+a}}=t $ thì t thuộc (0;1]

Bài toán đưa về tìm Min của: $f(t)=\frac{2}{\sqrt{t^2+1}}+\frac{t}{\sqrt{2}}+\frac{\sqrt{2}(t^2+1)}{2t+1}, 0

Ta có: $f'(t)=-2t(t^2+1)^{-3/2}+\frac{1}{\sqrt{2}}+2\sqrt{2}(t^2+t-2)(2t+1)^{-2}$ và $f''(t)=2(2t^2-1)(t^2+1)^{-5/2}+18\sqrt{2}(2t+3)^{-3}$

Dễ thấy rằng $f"(t)>0$ với mọi t thuộc (0;1]

Do đó f'(t) có duy nhất nghiệm t=1.

Khảo sát ta được $Minf(t)=f(1)=\frac{5\sqrt{2}}{2}$

Vậy Min P = $\frac{5\sqrt{2}}{2}$. Đẳng thức xảy ra khi a=b=c




#654896 Topic: [LTDH] Mỗi ngày hai bất đẳng thức.

Đã gửi bởi MrS on 20-09-2016 - 18:26 trong Bất đẳng thức và cực trị

Tiếp theo:

Bài 47: Cho $a,b,c$ là các số thực dương thỏa mãn: $(3a+2b+c)(\frac{1}{a}+\frac{2}{b}+\frac{3}{c})=30$. Tìm giá trị lớn nhất của biểu thức:

$P=\frac{b+2c-7\sqrt{72a^2+c^2}}{a}$.

Bài 48: Cho $a,b,c$ là các số thực dương. Tìm giá trị nhỏ nhất của:

$P=\sqrt{\frac{a}{b+c}}+\sqrt{\frac{b}{c+a}}+\sqrt{\frac{c}{b+a}}+\frac{\sqrt{2}(a+b+c)}{\sqrt{ab}+\sqrt{bc}+\sqrt{ca}}$.

Bài 48:

Không mất tổng quát giả sử c=min(a, b ,c).

Trước tiên ta CM bổ đề quen thuộc: $\sqrt{\frac{a}{b+c}}+\sqrt{\frac{b}{c+a}}\geq 2\sqrt{\frac{a+b}{a+b+2c}}$ ( có thể dùng Holder để chứng minh)

Lại theo AM - GM: $\sqrt{ab}+\sqrt{bc}+\sqrt{ca}\leq \frac{a+b}{2}+\sqrt{2c(a+b)}$

Đặt: $\sqrt{\frac{2c}{b+a}}=t$ với t thuộc (0;1]

Bài toán đưa về tìm Min của: $f(t)=\frac{2}{\sqrt{t^2+1}}+\frac{t}{\sqrt{2}}+\frac{\sqrt{2}(t^2+1)}{2t+1}$

Ta có: $f'(t)=-2t(t^2+1)^{-3/2}+\frac{1}{\sqrt{2}}+2\sqrt{2}(t^2+t-2)(2t+1)^{-2}$ và $f''(t)=2(2t^2-1)(t^2+1)^{-5/2}+18\sqrt{2}(2t+3)^{-3}$

Dễ thấy rằng $f"(t)>0$ với mọi với t thuộc (0;1]

Do đó f'(t) có duy nhất nghiệm t=1.

Khảo sát ta được $Minf(t)=f(1)=\frac{5\sqrt{2}}{2}$

Vậy Min P = $\frac{5\sqrt{2}}{2}$. Đẳng thức xảy ra khi a=b=c




#654866 CMR: $\sum \frac{(b+c-a)^2}{(b+c)^2+a^2}...

Đã gửi bởi MrS on 20-09-2016 - 13:49 trong Bất đẳng thức và cực trị

 

Cho $a,\,b,\,c$ là ba số thực sao cho $a^2 + b^2 + c^2 > 0.$ Chứng minh rằng

$$\frac{(b+c-a)^2}{(b+c)^2+a^2}+\frac{(c+a-b)^2}{(c+a)^2+b^2}+\frac{(a+b-c)^2}{(a+b)^2+c^2}\geq \frac{3}{5}.$$

 

Chuẩn hóa a+b+c =3, BĐT tương đương với:

$\sum \frac{1}{2a^2-6a+9}\leq \frac{3}{5}$

Dễ thấy: $\frac{1}{2x^2-6x+9}\leq \frac{2x+3}{25}$ với mọi $x\geq \frac{-1}{2}$ (nó tương đương với: $\frac{2(x-1)^2(2x+1)}{2x^2-6x+9}\geq 0$, mọi $x\geq \frac{-1}{2}$ )

Giả sử: $a\geq b\geq c\Rightarrow a\geq 1\geq c$

TH1: Nếu $a\geq b\geq c\geq \frac{-1}{2}$ thì $VT\leq \frac{2\sum a+9}{25}=\frac{3}{5}$

TH2: Nếu $c< \frac{-1}{2}$, dễ thấy $f(c)=\frac{1}{2c^2-6c+9}$ đồng biến với  $c< \frac{-1}{2}$ nên $f(c)< f(\frac{-1}{2})=\frac{2}{25}$

Lại có $\frac{1}{2a^2-6a+9}+\frac{1}{2b^2-6b+9}=\frac{1}{2(a-3/2)^2+9/2}+\frac{1}{2(b-3/2)^2+9/2}< \frac{1}{9/2}+\frac{1}{9/2}=\frac{4}{9}$

Từ đó suy ra $VT< \frac{2}{25}+\frac{4}{9}=\frac{118}{225}<\frac{3}{5}$

Từ các TH suy ra Đpcm.